Of the following hypothetical reforms in trial procedure, which one would the author be most likely to support as the...

dkl6bg on October 10 at 06:37PM

Why is the answer D and not E?

I was stuck between D and E and chose E because I felt like it was partially mentioned in the last few sentences of the passage. Why is the answer D?

Reply
Create a free account to read and take part in forum discussions.

Already have an account? log in

Emil-Kunkin on October 14 at 03:18AM

I think the issue with E is that the passage seems to be against the way judges deploy research, not with how lawyers do so. This last paragraph actually directly supports D. It tells us that judges are fully capable of minimizing inferential error in their jury instructions, but they are simply really bad at it. From this we can infer that if we trained them on how to give better jury instructions, they would be better at minimizing inferential error.